1. PYQ Topic wise - Number System
1. PYQ Topic wise - Number System
1. PYQ Topic wise - Number System
(CSAT 2024)
How many consecutive zeros are there at the end of the integer obtained in the product
𝟏𝟐 × 𝟐𝟒 × 𝟑𝟔 × 𝟒𝟖 × … × 𝟐𝟓𝟓𝟎 ?
(a) 50
(b) 55
(c) 100
(d) 200
Solution (d):
The result of a product will have a zero when we can write product as a multiple of 10. A 10 in a product is
obtained when 2 and 5 are multiplied. The number of zeroes will depend on whichever is less in number from
among 2 and 5.
In the given product 12 × 24 × 36 × 48 × … .× 2550 , we can observe that 2 is definitely more (as every
second number in product is a multiple of 2) in number than 5. So, the number of zeroes at the end will be
equal to the number of 5’s in the given product.
The number of 5’s in the given expression is:
510 = 10
1020 = 220 × 520 = 20
1530 = 330 × 530 = 30
2040 = 440 × 540 = 40
2550 = 5100 = 100
So, the number of zeroes at the end of the product = 10 + 20 + 30 + 40 + 100 = 200.
(CSAT 2021)
If 32019 is divided by 10, then what is the remainder?
(a) 1
(b) 3
(c) 7
(d) 9
Solution (c):
Remainder obtained when a number is divided by 10 and unit digit of the number is same value.
So, remainder when 32019 is divided by 10 and unit digit of 32019 is same number.
We know that, unit digit of 34𝑛 (where 4n is any multiple of 4) is 1.
Now, on dividing 2019 by 4, we get a remainder 3 .Hence, 32019 will have the same last digit as that of 33 i.e. 7.
So, remainder will be 7.
(CSAT 2024)
What is the rightmost digit preceding the zeros in the value of 𝟑𝟎𝟑𝟎 ?
(a) 1
(b) 3
(c) 7
(d) 9
Solution (d):
3030 = (10 × 3)30 =1030 × 330.
Since, 1030 will result in a 31-digit number with 30 zeros at the end. So, the rightmost digit preceding
the zeros will be the unit digit of 330 .
We know that, unit digit of 34𝑛 (where 4n is any multiple of 4) is 1.
Now, on dividing 30 by 4, we get a remainder 2 .Hence, 330 will have the same last digit as that of 32 i.e. 9.
(CSAT 2019)
In a school every student is assigned a unique identification number. A student is a football player if and only
if the identification number is divisible by 4, whereas a student is a cricketer if and only if the identification
number is divisible by 6. If every number from 1 to 100 is assigned to a student, then how many of them play
cricket as well as football?
(a) 4
(b) 8
(c) 10
(d) 12
Solution (b):
If a student has to play both cricket and football, then his/her identification number must be divisible by both 4
and 6.
LCM of (4, 6) = 12 .There are 8 multiples of 12 between 1 to 100 which are 12, 24, 36, 48, 60, 72, 84, 96. There
are 8 students who play both cricket and football.
(CSAT 2024)
𝟐𝟐𝟐𝟑𝟑𝟑 + 𝟑𝟑𝟑𝟐𝟐𝟐 is divisible by which of the following numbers ?
(a) 2 and 3 but not 37
(b) 3 and 37 but not 2
(c) 2 and 37 but not 3
(d) 2, 3 and 37
Solution (b):
222333 + 333222 = [(222)3 ]111 + [(333)2 ]111
= [(23 × 1113 )]111 + [(32 × 1112 )]111
= [(8 × 1113 )111 + [(9 × 1112 )]111
= (1112 )111 × [{(8 × 111)111 + (9)111 }]
= 111222 × [888111 + 9111 ]
= 111 × (111)221 × [888111 + 9111 ]
Now, we know that 111 = 37 × 3. Also, the expression 111221 × [888111 + 9111 ] is definitely odd.
Therefore, we can say that the given expression is divisible by 37 and 3, but not 2.
(CSAT 2016)
If R and S are different integers both divisible by 5, then which of the following is not necessarily true?
(a) R – S is divisible by 5
(b) R + S is divisible by 10
(c) R x S is divisible by 25
(d) 𝐑𝟐 + 𝐒 𝟐 is divisible by 5
Solution (b):
Given that both R and S are different integers divisible by 5.
a) R - S is divisible by 5: This is necessarily true because when we subtract two numbers that are divisible by 5,
the result is also divisible by 5.
b) R+S is divisible by 10: This is not necessarily true because one of the numbers could be 5 and the other could
be 10, both of which are divisible by 5 but their sum is not divisible by 10.
c) R x S is divisible by 25: This is necessarily true because when we multiply two numbers that are divisible by 5,
the result is also divisible by 25.
d) R2 + S 2 is divisible by 5: This is necessarily true because when we add two numbers that are divisible by 5,
the result is also divisible by 5.
(CSAT 2020)
Let XYZ be a three-digit number, where (X + Y + Z) is not a multiple of 3. Then (XYZ + YZX + ZXY) is not
divisible by
(a) 3
(b) 9
(c) 37
(d) (X + Y + Z)
Solution (b):
XYZ is a three-digit number.
So, we can write this as: 100X + 10Y + Z (as X is at hundreds place and Y is at tens place)
So, XYZ + YZX + ZXY = (100X + 10Y + Z) + (100Y + 10Z + X) + (100Z + 10X + Y) = 111 (X + Y + Z)
Hence, it is divisible by (X + Y + Z).
Also, 111 is divisible by 3, as well as 37.
So, the expression XYZ + YZX + ZXY is not divisible by 9.
(CSAT 2024)
𝟑𝟐𝟓 + 𝟐𝟐𝟕 is divisible by
(a) 3
(b) 7
(c) 10
(d) 11
Solution (c):
325 + 227 = (25 )5 + 227
=225 + 227
= 225 (1 + 22 )
= 225 (1 + 4)
= 224 × 2(5)
= 224 × 10.
So , 325 + 227 is divisible by 10.
(CSAT 2020)
How many integers are there between 1 and 100 which have 4 as a digit but are not divisible by 4?
(a) 5
(b) 11
(c) 12
(d) 13
Solution (c):
The integers between 1 and 100 which have 4 as a digit are:
4, 14, 24, 34, 40, 41, 42, 43, 44, 45, 46, 47, 48, 49, 54, 64, 74, 84 and 94
So, there are a total 19 such integers.
Out of these, the integers which are divisible by 4 are:
4, 24, 40, 44, 48, 64 and 84
So, the number of integers not divisible by 4 = 19 – 7 = 12 integers
Hence, option (c) is the correct answer.
(CSAT 2021)
The number 3798125P369 is divisible by 7, what is the value of the digit P?
(a) 1
(b) 6
(c) 7
(d) 9
Solution (b):
Using divisibility rule of 7
(CSAT 2019)
An 8-digit numbers 4252746B leaves remainder 0 when divided by 3. How many values of B are possible ?
(a) 2
(b) 3
(c) 4
(d) 6
Solution (c):
Given numbers = 4 2 5 2 7 4 6 B
Sum of digits = 4 + 2 + 5 + 2 + 7 + 4 + 6 + B = 30 + B
We know that a number whose sum of digits is divisible by 3 is also divisible by 3.
Hence, the possible values of B can be 0, 3, 6 or 9.
(CSAT 2022)
An Identity Card has the number ABCDEFG, not necessarily in that order, where each letter represents a
distinct digit (1, 2, 4, 5, 7, 8, 9 only). The number is divisible by 9. After deleting the first digit from the right,
the resulting number is divisible by 6. After deleting two digits from the right of original number, the
resulting number is divisible by 5. After deleting three digits from the right of original number, the resulting
number is divisible by 4. After deleting four digits from the right of original number, the resulting number is
divisible by 3. After deleting five digits from the right of original number, the resulting number is divisible by
2. Which of the following is a possible value for the sum of the middle three digits of the number?
(a) 8
(b) 9
(c) 11
(d) 12
Solution (a):
The number has 7 digits, and has been denoted by: ABCDEFG
These letters needed to be replaced by 1, 2, 4, 5, 7, 8, 9, not necessarily in the same order.
We have to find the possible value of C + D + E
The original number (ABCDEFG) is divisible by 9. It has to be as 1 + 2 + 4 + 5 + 7 + 8 + 9 = 36, which is divisible by
9. This information is utterly useless.
After deleting 1 digit from the right, the resulting number (ABCDEF) is divisible by 6 (means number should be
even as well as divisible by 3). It means that, F = 2, 4 or 8 (i.e. an even number).
Also, if even after removing G, the remaining number is divisible by 3, then it means G = 9.
After deleting 3 digits from the right, the resulting number (ABCD) is divisible by 4. It means that, D = 2, 4 or 8
(i.e. an even number).
After deleting 5 digits from the right, the resulting number (AB) is divisible by 2. It means that, B = 2, 4 or 8 (i.e.
an even number).
So, F, D and B are even numbers (2, 4 or 8). And, A, C, E, and G are odd numbers (1, 5, 7 or 9).
After deleting 2 digits from the right, the resulting number (ABCDE) is divisible by 5. It means that, E = 5.
So, we just have to find C + D + E = C + D + 5, which must be an even number as C is odd (1, or 7), and D is even
(2, 4, or 8).
On observing the options, we can see that C must be 1 and D must be 2. So, C + D + E = 1 + 2 + 5 = 8.
Note: This information was also redundant - After deleting 4 digits from the right, the resulting number (ABC) is
divisible by 3. But it may be useful if we want to know the entire number. The only odd number remaining is 7.
So, A = 7. So, the ABC is actually 7B1. B can be 4 or 8. But for 7B1 to be divisible by 3, B must be 4. So, F = 8. So,
the seven-digit number is 7412589.
(CSAT 2021)
Consider all 3-digit numbers (without repetition of digit) obtained using three non-zero digits which are
multiples of 3. Let S be their sum. Which of the following is/are correct?
1. S is always divisible by 74.
2. S is always divisible by 9.
(a) Only 1
(b) Only 2
(c) Both 1 and 2
(d) Neither 1 nor 2
Solution (c):
Three non-zero digits which are multiples of 3 are 3,6 and 9 .
Using these 3 digits, we can make 3! i.e. 6 three numbers.
So, their sum (s) = 369 + 396 + 639 +693+ 936+963 = 3996 .
3996 is divisible by both 74 and 9.
(CSAT 2023)
What is the remainder if 2192 is divided by 6
(a) 0
(b) 1
(c) 2
(d) 4
Solution (d):
2192 2191
Method I: 6 = 3
Remainder when 22 is divided by 3 = 1
Remainder when 2191 i.e. {(22 )95 × 2} is divided by 3 = Remainder when {(12 )95 × 2} is divided by 3 =2.
2192
So, Remainder [ 6 ] = 2 × 2 = 4
Method II:
To find the remainder when 2192 is divided by 6, we can observe a pattern in the remainders of powers of 2
when divided by 6.
When we divide powers of 2 by 6, we get the following sequence of remainders:
21 divided by 6 leaves a remainder of 2.
22 divided by 6 leaves a remainder of 4.
23 divided by 6 leaves a remainder of 2.
24 divided by 6 leaves a remainder of 4.
25 divided by 6 leaves a remainder of 2.
We notice that when power of 2 is odd, the remainder is 2; and when power of 2 is even, the remainder is 4.
Therefore, the answer would be 4.
(CSAT 2020)
What is the remainder when 51 × 27 × 35 × 62 ×75 is divided by 100 ?
(a) 50
(b) 25
(c) 5
(d) 1
Solution (a):
The expression is: 51 × 27×35 × 62 ×75
51 × 27 ×35 × 62 × 75 51 × 27 × 35 × 62 ×3 (51 × 27 × 35 ×31 × 3)
100
= 4
= 2
So, we need to find the remainder when 51 × 27 × 35 × 31 × 3 is divided by 2.
The value of 51 × 27 ×35 × 31 × 3 must be an odd number, as it is a multiplication of all odd numbers. Hence,
when it is divided by 2, we will get 1 as remainder.
But we cancelled out 50 earlier.
So, the remainder when 51 × 27 × 35 × 62 × 75 is divided by 100 = 1 × 50 = 50
(CSAT 2023)
What is the remainder when 85 × 87 × 89 × 91 × 95 × 96 is divided by 100.
(a) 0
(b) 1
(c) 2
(d) 4
Solution (a):
In 85 × 87 × 89 × 91 × 95 × 96 there are two 5’s (in 85 and 95) and a 4 (in 96).
We know that, 5 × 5 × 4 = 100
So, the given expression is completely divisible by 100. So, remainder will be zero.
(CSAT 2023)
A number N is formed by writing 9 for 99 times. What is the remainder if N is divided by 13 ?
(a) 11
(b) 9
(c) 7
(d) 1
Solution (a):
Method I: N = 9999 …… 99 times
Any digit repeated (P - 1) times is divisible by P, where P is a prime number > 5.
So, 9999 …… repeated 13 - 1 = 12 times will be divisible by 13. So, 9999 …… repeated 12 × 8 = 96 times will be
9999…… 96 𝑡𝑖𝑚𝑒𝑠
divisible by 13. That is, Remainder [ 13
] =0
9999…… (96 𝑡𝑖𝑚𝑒𝑠)000
Or Remainder [ 13
] =0
999
So, we just need to find out the remainder when we divide the remaining three digits by 13. Remainder [ ] =
13
11
Hence, option (a) is correct.
Method II:
We can analyze the pattern of remainders. Remainder [9/13] = 9
Remainder [99/13] = 8
Remainder [999/13] = 11
Remainder [9999/13] = 2
Remainder [99999/13] = 3
Remainder [999999/13] = 0
This pattern can be seen getting repeated thereafter too. Remainder [9999999/13] = 9
Remainder [99999999/13] = 8 …and so forth.
So, if total number of 9s is six, twelve, eighteen, ……., ninety, ninety-six, etc., the remainder is 0.So, if the
number has ninety-seven 9s, the remainder is 9. [Following the pattern] So, if the number has ninety-eight 9s,
the remainder is 8.So, if the number has ninety-nine 9s, the remainder is 11. Therefore, the answer is 11.
(CSAT 2017)
A 2-digit number is reversed. The larger of the two numbers is divided by the smaller one. What is the largest
possible remainder ?
(a) 9
(b) 27
(c) 36
(d) 45
Solution (d):
The goal is to maximize the remainder when dividing two two-digit numbers. To do this, one of the numbers
should be as large as possible, so one of its digits must be 9, creating numbers in the form 9X (where X is the
unit’s digit) and X9 (where X is the ten’s digit).
To achieve the largest remainder : The smaller number should not divide the larger one more than once;
otherwise, the remainder will be too small. If the smaller number is too large, it will reduce the remainder, as
the difference between the two numbers will be minimal.
Examples:
When x = 1 , the numbers are 91 and 19, and dividing 91 by 19 gives a remainder of 15.
When x = 8 , the numbers are 98 and 89, and the remainder is 9.
To find the optimal solution, we try mid-range values for x :
(CSAT 2023)
What is the sum of all digits which appear in all the integers from 10 to 100 ?
(a) 855
(b) 856
(c) 910
(d) 911
Solution (b):
We have to find the sum of all the digits of the numbers from 10 to 100.
Let’s leave aside 100 for now. We are left with 9 sets of 10 numbers each.
10, 11, ….. 19
20, 21, ….. 29
90, 91, ….. 99
Counting Units Digits
9 × 10
Sum of unit digits of each of these 9 sets = 0 + 1 + 2 + …. + 9 = 2 = 45
𝑛(𝑛 + 1)
[ Sum of first n natural numbers = 2
]
(CSAT 2019)
The number of times the digit 5 will appear while writing the integers from 1 to 1000 is
(a) 269
(b) 271
(c) 300
(d) 302
Solution (c):
The frequency of occurrence of digit 5 in different series of numbers are as follows
From 1 to 10 → 1 time
11 to 2→1 time
21 to 30 → 1 time
31 to 40 →1 time
41 to 50 → 2 times
51 to 60→10 times
61 to 70 →1 time
71 to 80 → 1 time
81 to 90 → 1 time
91 to 100 → 1 time
Occurrence of 5 from 1 to 100 = 20 times. Similarly, occurrence of 5 from 1 to 1000 = 20 x 10 = 200 times
But occurrence of 5 at hundredth place from 500 to 600 = 100 times
Total number of times digit 5 would appear = 200 + 100 = 300 times
(CSAT 2019)
A printer numbers the pages of a book starting with 1 and uses 3089 digits in all. How many pages does the
book have?
(a) 1040
(b) 1048
(c) 1049
(d) 1050
Solution (c):
The single-digit page (From 1 to 9) = 9 digit .
Double digit page (From 10 to 99) = 2 x 90 = 180 digit
Three-digit page (From 100 to 999) = 3 x 900 = 2700 digit
Total digit used = 9 + 180 + 2700 = 2889.
200
The remaining digit = 3089 - 2889 = 200. The number of pages with four digits = 4 = 50.
The required number of pages = 999 + 50 =1049.
(CSAT 2017)
What is the total number of digits printed, if a book containing 150 pages is to be numbered from 1 to 150 ?
(a) 262
(b) 342
(c) 360
(d) 450
Solution (b):
Digits from page 1 to page 9 (total pages)= 9×1=9.
Digits from page 10 to page 99 (total pages) = 90×2=180
Digits from page 100 to page 150 (total pages 51) =51×3 =153
Total number of digits from page 1 to page 150 = 9 + 180 + 153 = 342
(CSAT 2024)
Consider the following statements in respect of the sum S = x + y + z, where x, y and z are distinct prime
numbers each less than 10 :
1. The unit digit of S can be 0.
2. The unit digit of S can be 9.
3. The unit digit of S can be 5.
Which of the statements given above are correct?
(a) 1 and 2 only
(b) 2 and 3 only
(c) 1 and 3 only
(d) 1, 2 and 3
Solution (c):
x, y and z are distinct prime numbers less than 10. So, these numbers can be 2, 3, 5, or 7.
S=x+y+z
S = 2 + 3 + 5 = 10 ………..(1)
S = 3 + 5 + 7 = 15 …………(3)
So, (2,3,5) and (3,5,7) satisfy conditions 1 and 3 respectively. However, statement 2 is not true for these
given set of numbers.
(CSAT 2024)
Let p, q, r and s be distinct positive integers. Let p, q be odd and r, s be even. Consider the following
statements :
1. (𝐩 − 𝐫)𝟐 (𝐪𝐬) is even.
2. (𝐪 − 𝐬)𝐪𝟐 𝐬 is even.
3. (𝐪 + 𝐫)𝟐 (𝐩 + 𝐬) is odd.
Which of the statements given above are correct ?
(a) 1 and 2 only
(b) 2 and 3 only
(CSAT 2024)
Let p and q be positive integers satisfying p < q and p+q=k . What is the smallest value of k that does not
determine p and q uniquely?
(a) 3
(b) 4
(c) 5
(d) 6
Solution (c):
it is given that p and q are positive integers, such that p<q.
p+q=k
Case 1: If k = 3
p+q=3
1+2=3
Here, p and q have unique values.
Hence, this is discarded.
Case 2: If k = 4
p+q=4
1+3=4
Here, p and q have unique values.
Hence, this is discarded.
Case 3: If k = 5
p+q=5
1+4=5
2+3=5
Thus, here p can be 1 or 2 and q can be 4 or 3.So, here p and q have multiple possible values.
(CSAT 2020)
For what value of n, the sum of digits in the number (𝟏𝟎𝒏 + 1) is 2?
(a) For n = 0 only
(b) For any whole number n
(c) For any positive integer n only
(d) For any real number n
Solution (b):
Given number, N = (10𝑛 + 1)
On putting n = 0, we get:
𝟏𝟎𝒏 + 1 = 100 + 1 = 1 + 1 = 2 {sum of digits is 2}
On putting n = 1, we get:
𝟏𝟎𝒏 + 1 = 101 + 1 = 10 +1= 11 {sum of digits is 2}
On putting n = 2, we get:
𝟏𝟎𝒏 + 1 = 102 + 1 = 100 + 1 = 101 {sum of digits is 2}
On putting n = 3, we get:
𝟏𝟎𝒏 + 1 = 103 + 1 = 1000 + 1 = 1001 {sum of digits is 2}
Hence, sum of digits of number N will always be 2 if n = 0, 1, 2, 3 ...
Or we can say that the sum of digits of number N will always be 2 if n is any whole number.
(CSAT 2023)
Three of the five positive integers p, q, r, s, t are even and two of them are odd (not necessarily in order).
Consider the following:
1. p + q + r – s - t is definitely even.
2. 2p + q + 2r - 2s + t is definitely odd.
Which of the above statements is/are correct?
(a) 1 Only
(b) 2 Only
(c) Both 1 and 2
(d) Neither 1 nor 2
Solution (a):
Odd-even integers: p, q, r, s, t. We know that the sum or difference of two even or two odd numbers is always
even. The sum/difference of any number of even numbers is even.
Statement 1:
p, q, r, s, t are five integers: three even and two odd.
The sum or difference of three even numbers minus/plus the sum/difference of two odd numbers results in
even - even, which is always even.
Thus, Statement 1 is correct.
Statement 2:
2p + q + 2r – 2s + t can be rearranged to 2(p + r – s) + (q + t).
2(p + r – s) is even.
Thus, Statement 2 depends on the values of q and t and is not definitely odd.
Only Statement 1 is correct.
(CSAT 2023)
Consider the following in respect of prime number p and composite number c.
1. p+c/p-c can be even.
2. 2p+c can be odd.
3. pc can be odd.
Which of the statements given above are correct?
(a) 1 and 2 only
(b) 2 and 3 only
(c) 1 and 3 only
(d) 1, 2 and 3
Solution (d):
Let’s place some numbers in place of p and c in the given expressions and check them out.
Statement 1:
𝑝 + 𝑐 11 + 9 20
=
𝑝 − 𝑐 11 – 9
= 2 = 10 (an even number) So, statement 1 is correct.
Statement 2:
2p + c = (2×3) + 9 = 15 (an odd number) So, statement 2 is correct.
Statement 3:
pc = 3×9 = 27 (an odd number) So, statement 3 is correct.
(CSAT 2022)
Consider the following statements in respect of two natural numbers p and q such that p is a prime number
and q is a composite number :
1. pxq can be an odd number.
2. q/p can be a prime number.
3. p+q can be a prime number.
Which of the above statements are correct?
(a) 1 and 2 only
(b) 2 and 3 only
(c) 1 and 3 only
(d) 1, 2 and 3
Solution (d):
p is a prime number. So, p can be 2, 3, 5, 7, 11, 13, .….
q is a composite number. So, q can be 4, 6, 8, 9, 10, .….
Statement 1: p × q can be an odd number, e.g. (3 × 9 = 27). Thus, statement 1 is correct.
Statement 2: q/p can be a prime number, e.g. (4/2 = 2). Thus, statement 2 is correct.
Statement 3: p + q can be a prime number, e.g. (3 + 4 = 7). Thus, statement 3 is correct.
(CSAT 2019)
How many triplets (x, y, z) satisfy the equation x + y + z = 6, where x, y and z are natural numbers?
(a) 4
(b) 5
(c) 9
(d) 10
Solution (d):
There are three possible values of triplets satisfying x + y + z = 6
which are (1, 2, 3),(1, 1, 4) and (2, 2, 2)
Now, number of arrangements of 1, 2, 3 are 3! (because all 3 digits are different) = 3x2×1 = 6 .
3!
Number of arrangements of 1, 1, 4 are = (as two numbers are same out of three numbers) = 3.
2!
Number of arrangements of 2, 2, 2 are 1 (because all three digits are same).
Total possible triplets are 6 + 3+ 1 = 10.
(CSAT 2020)
How many pairs of natural numbers are there such that the difference of whose squares is 63?
(a) 3
(b) 4
(c) 5
(d) 2
Solution (a):
Let the required pair of natural number is x and y.
As per the question,
X 2 – y 2 = 63
or (x + y) (x – y) = 63
There are three possible cases in which product of two numbers is 63.
Case 1 :
(x + y) = 9 and (x – y) = 7
Then x = 8 and y = 1
Case 2 :
(x + y) = 21 and (x – y) = 3
Then x = 12 and y = 9
Case 3 :
(x + y) = 63 and (x – y) = 1
Then x = 32 and y = 31
Hence, there are three pairs of natural numbers such that the difference of their squares is 63.
(CSAT 2023)
How many natural numbers are there which given a remainder of 31 when 1186 is divided by these natural
numbers?
(a) 6
(b) 7
(c) 8
(d) 9
Solution (d):
The remainder when a number is divided by another is the difference between the dividend and the largest
multiple of the divisor that does not exceed the dividend.
Here, the remainder is 31 when 1186 is divided by a certain number, meaning the divisor must be a factor of
1186 - 31 = 1155.
The divisors of 1155 are 1, 3, 5, 7, 11, 15, 21, 33, 35, 55, 77, 105, 165, 231, 385, 1155.
Since the remainder is 31, the divisor must be greater than 31. Therefore, the possible divisors are 33, 35, 55,
77, 105, 165, 231, 385, and 1155.Thus, there are 9 such numbers.
(CSAT 2019)
If x is greater than or equal to 25 and y is less than or equal to 40, then which one of the following is always
correct?
(a) x is greater than y.
(b) (y-x) is greater than 15.
(c) (y-x) is less than or equal to 15.
(d) (x-y) is greater than or equal to 65.
Solution (c):
According to question ,
x ≥ 25 ⇒ x = {25, 26, 27,----). and y ≤ 40 ⇒ y = (40,39,38………..).
For (y - x) maximum, y should be maximum and x should be minimum. ( y- x) ≤ (40 - 25) .
∴ (y - x) ≤ 15. Hence, (y - x) is away less than or equal to 15.
(CSAT 2020)
Let p, q, r and s be natural numbers such that
p – 2016 = q + 2017 = r – 2018 = s + 2019
Which one of the following is the largest natural number?
(a) p
(b) q
(c) r
(d) s
Solution (c):
Method I:
In questions like this, we need to keep in mind that we are just supposed to compare the numbers and so we
should avoid wasting our time in finding their real values.
p – 2016 = q + 2017 which means p = q + 2017 + 2016 (it means p > q)
q + 2017 = r – 2018 or r = q + 2017 + 2018 (it means r > q)
r – 2018 = s + 2019 or r = s + 2019 + 2018 (it means r > s)
p – 2016 = r – 2018 or r = p – 2016 + 2018 = p + 2 (it means r > p)
We already know that r > q, s . Hence, r is the largest number.
Method II:
Let s = 0. So, we get: p – 2016 = q + 2017 = r – 2018 = 2019
So, p – 2016 = 2019 Or p = 2019 + 2016 = 4035
And q + 2017 = 2019 or q = 2019 – 2017 = 2
And r – 2018 = 2019 Or r = 2019 + 2018 = 4037
Hence, r is the largest number.
(CSAT 2020)
What is the largest number among the following?
𝟏 −𝟔
(a) ( )
𝟐
𝟏 −𝟑
(b) (𝟒)
𝟏 −𝟒
(c) (𝟑)
𝟏 −𝟐
(d) ( )
𝟔
Solution (c):
1 –6
Option (a): (2) = 26 = 64
1 –3
Option (b): (4) = 43 = 64
1 –4
Option (c): (3) = 34 = 81
1 –2
Option (d): ( ) = 62 = 36
6
We can see that 81 is the largest number among the given four numbers.
(CSAT 2018)
X and Y are natural numbers other than 1, and Y is greater than X. Which of the following represents the
largest number?
(a) XY
(b) X/Y
(c) Y/X
𝐗+𝐘
(d) 𝐗𝐘
Solution (a):
Given X and Y are natural numbers i.e. (1, 2, 3, … N)
Given, X, Y 1 and Y > X.
Let, us assume X = 2, Y = 4
For these values, we get,
(a) XY = 2 4 = 8
X 2
(b) Y = 4 = 0.5
Y 4
(c) = = 2
X 2
X+Y 6 3
(d) = = = 0.75
XY 8 4
Option (a) i.e., XY will be greatest number.
(CSAT 2019)
If the numerator and denominator of a proper fraction are increased by the same positive quantity which is
greater than zero, the resulting fraction is
(a) always less than the original fraction
(b) always greater than the original fraction
(c) always equal to the original fraction
(d) such that nothing can be claimed definitely
Solution (b):
1
Let the fraction be 2.
Suppose we add a number 3 to both numerator and denominator, we get
1+3 3
=
2+3 5
1 3
Now, 2 = 0.5 = 5 = 0.6
3 1
>
5 2
Hence, the new fraction is always greater than the original fraction.
(CSAT 2017)
There are certain 2-digit numbers. The difference between the number and the one obtained on reversing it
is always 27. How many such maximum 2-digit numbers are there?
(a) 3
(b) 4
(c) 5
(d) None of these
Solution (d):
Let the numbers be of the form 10x + y. where x is ten’s digit and y unit’s digit.
Number obtained after reversing digits = (10y + x).
Then, 10x + y – (10y + x) = 9x – 9y = 9 (x – y)
Now, difference is 27 when x – y = 3
So, required numbers are 14, 25, 36, 47, 58, 69, 41, 52, 63, 74, 85 and 96.
(CSAT 2017)
Certain 3-digit numbers have the following characteristics:
1. All the three digits are different.
2. The number is divisible by 7.
3. The number on reversing the digits is also divisible by 7.
How many such 3-digit numbers are there?
(a) 2
(b) 4
(c) 6
(d) 8
Solution (b): Let the number "100a + 10b + c" where a, b, and c is the single digit number (a > 0).Now the
reverse number shall be "100c + 10b + a’. Both numbers are divisible by 7 hence their difference also divisible
by 7.
(CSAT 2021)
The difference between a 2-digit number and the number obtained by interchanging the positions of the
digits is 54. Consider the following statements
(1) The sum of the two digits of the number can determined only if the product of the two digits is known.
(2) The difference between the two digits of the number can be determined.
Which of the above statements is/are correct?
(a) 1 only
(b) 2 only
(c) Both 1 and 2
(d) Neither 1 nor 2
Solution (b):
Let's say the 2-digit number is 10x + y where x and y are the tens and units’ digits respectively. When we interchange
the positions of the digits, we get a new number: 10y+ x. The difference between these two numbers is given as
54.So, we have:
(10x + y) - (10y + x) = 54
9x - Sy = 54
x-y=6
From this equation, we can determine that the difference between the two digits of the number is 6. So, statement
2 is correct.
However, knowing only that x - y = 6 does not allow us to determine the sum of the two digits without additional
information. So, statement 1 is incorrect.
(CSAT 2022)
Let p be a two-digit number and q be the number consisting of same digits written in reverse order.
If p x q=2430, then what is the difference between p and q?
(a) 45
(b) 27
(c) 18
(d) 9
Solution (d):
p is a two-digit number, and q is the number consisting of the same digits in the reverse order.
It’s given that, p × q = 2430.
The last digit of the product is 0, which indicates that one two of the digits must be 5 and other should be even.
So, possibilities for p are 50,52,54,56,58 .Now using given conditions and possibilities of p, we can find out if p
is 54 then q has to be 45 so that p × q = 2430.Hence difference is 9.
(CSAT 2018)
A number consists of three digits of which the middle one is zero and their sum is 4. If the number formed by
interchanging the first and last digits is greater than the number itself by 198, then the difference between
the first and last digits is
(a) 1
(b) 2
(c) 3
(d) 4
Solution (b) :
Let, the given number be (100x+ y).After changing it would be = (100y + x)
Given, (100y+x) - (100x+y) = 198
99 y – 99 x = 198
y – x = 2. Difference between first and last digit is 2.
(CSAT 2020)
One page is torn from a booklet whose pages are numbered in the usual manner starting from the first page
as 1. The sum of the numbers on the remaining pages is 195. The torn page contains which of the following
numbers.
(a) 5, 6
(b) 7, 8
(c) 9, 10
(d) 11, 12
Solution (b) :
𝑛(𝑛+1)
Let the number of pages in the book be sum of consecutive numbers from 1 to n = = around 195 or
2
n(n+1) = around 390
As a page with two numbers was torn, so the value of n(n+1) must be more than 390.
The minimum possible value of n(n+1) over 390, such that n is an integer is got when we put n = 20
So, n(n+1) = 20 × 21 = 420
𝑛(𝑛+1) 20 ×21
So, sum of 20 pages = = = 210
2 2
Hence, sum of the two numbers on the torn page = 210 – 195 = 15. Only option (b) yields 15 as sum, i.e. 7 + 8 =
15.
(CSAT 2021)
When a certain number is multiplied by 7, the product entirely comprises ones only (1111…). What is the
smallest such number?
(a) 15713
(b) 15723
(c) 15783
(d) 15873
Solution (d):
As the number comprising all 1's is obtained on multiplied by 7, so it means that 7 is the factor of that number.
Our answer will be the smallest number comprising all 1's that will be divisible by 7.
111111
7
= 15873 .15873 is the smallest such number.
(CSAT 2023)
If p, q, r and s are distinct single digit positive numbers, then what is the greatest value of
(p +q)(r + s) ?
(a) 230
(b) 225
(c) 224
(d) 221
Solution (b) :
The greatest value of (p + q)(r + s) will be achieved when p, q, r, and s are the largest distinct single digit positive
numbers.
The largest distinct single digit positive numbers are 9, 8, 7 and 6.For these constant values
(p +q)(r + s) will be maximum if (p +q) = (r + s).
when we rearrange the numbers as (9 + 6)(8 + 7) = 15 * 15 = 225.
So, the correct answer is (b) 225.
(CSAT 2023)
D is a 3-digit number such that the ratio of the number to the sum of its digits is least. What is the difference
between the digit at the hundred's place and the digit at the unit's place of D?
(a) 0
(b) 7
(c) 8
(d) 9
Solution (c) :
To minimize the ratio of a number to the sum of its digits, we need to maximize the sum of the digits while
keeping the number as small as possible. Although 100 is the smallest 3-digit number, its digit sum is not
maximized. Instead, the smallest 3-digit number with the highest digit sum is 199 (since 1+9+9=19). This results
in a ratio of 10.47, the smallest achievable for a 3-digit number. The difference between the hundreds digit (1)
and the units digit (9) of 199 is 9-1=8.
(CSAT 2018)
If x – y = 8, then which of the following must be true?
1. Both x and y must be positive for any value of x and y
2. If x is positive, y must be negative for any value of x and y
3. If x is negative, y must be positive for any value of x and y
Select the correct answer using the code given below
(a) Only 1
(b) Only 2
(c) Both 1 and 2
(d) Neither 1 nor 2 nor 3
Solution (d):
Given x – y=8
Statement 1: Given x and y should be positive number for x – y = 8.
Assume x = 2, y = – 6
x – y=(2) – (– 6)=8 The given statement is wrong.
Statement 2: If x is positive and y must be negative.
Let x = 16, y = 8
x – y = 16 – 8 = 8The given statement is wrong.
Statement 3: If x is negative y must be positive.
Let x= – 2, y = - 10
x – y= – 2 – (-10) = 8 The given statement is wrong.
Hence, option (d) is correct answer.
(CSAT 2021)
Integers are listed from 700 to 1000. In how many integers ,is the sum of the digits 10?
(a) 6
(b) 7
(c) 8
(d) 9
Solution (d):
The sum of an integer's digits equals 10 if the digits add up to 10. For instance, the digit sum of 703 is
7 + 0 + 3 = 10. To find all integers between 700 and 1000 with a digit sum of 10, we can list them: 703, 712, 721,
730, 802, 811, 820, 901, 910. There are 9 such integers. Therefore, the correct answer is (d) 9.
(CSAT 2022)
What is the number of numbers of the form 0.XY, where X and Y are distinct non-zero digits?
(a) 72
(b) 81
(c) 90
(d) 100
Solution (a):
Since, X and Y are distinct non-zero digits ∴ The required number of numbers of the form 0.XY = 9 × 8 = 72
.Hence, option (a) is the correct answer.
(CSAT 2020)
The recurring decimal representation 1.272727 … is equivalent to
𝟏𝟑
(a) 𝟏𝟏
𝟏𝟒
(b)
𝟏𝟏
𝟏𝟐𝟕
(c) 𝟗𝟗
𝟏𝟑𝟕
(d) 𝟗𝟗
Solution (b):
Method I:
1
1.272727… can be written as 1 + 0.272727… = 1 + 3(0.909…) = 1 + 3(11) {since, 0.909…. = 1/11}
3 14
= 1 + (11) = 11
Hence, the correct answer is option (b).
Method II:
Let x = 1.272727….
Then 100x = 127.272727….
So, 100x – x = (127.272727….) – (1.272727….) = 126
126 14
Or x = 99 = 11
Hence, the correct answer is option (b).
(CSAT 2022)
Let A, B and C represent distinct non-zero digits. Suppose x is the sum of all possible 3-digit numbers formed
by A, B and C without repetition. Consider the following statements:
1. The 4-digit least value of x is 1332.
2. The 3-digit greatest value of x is 888.
Which of the above statements is/are correct?
(a) 1 only
(b) 2 only
(c) Both 1 and 2
(d) Neither 1 nor 2
Solution (a):
The three-digit numbers have been represented by ABC, wherein A, B, and C are non-zero digits.
Using 3 distinct digits we can make 3 × 2 × 1 = 6 three-digit numbers.
So, x will be the sum of these 6 three-digit numbers.
We need to find the two values of x closest to 1000, one just below it (which will be the greatest 3-digit value
of x), and the other just above it (which will be the lowest 4-digit value of x).
Now, we have to do a bit of hit and try, so that the value of x reaches close to 1000.
Let the three digits be the minimum possible ones, i.e. 1, 2, and 3.
So, we get x = 123 + 132 + 213 + 231 + 312 + 321 = 1332
This is the least possible value of x. So, statement 1 is correct, but statement 2 is incorrect.
(CSAT 2021)
Consider the following statements:
1. The sum of 5 consecutive integers can be 100.
2. The product of three consecutive natural numbers can be equal to their sum.
Which of the above statements is/are correct?
(a) 1 only
(b) 2 only
(c) Both 1 and 2
(d) Neither 1 nor 2
Solution (c):
From statement 1.Let the 5 consecutive numbers be x -2, x - 1, x, x+ 1 and x + 2
(x - 2) + (x - 1) + x + (x + 1) + (x + 2) = 100
100
5x = 100 = x= 5 = 20
So, the numbers are 18, 19, 20, 21 and 22.
.: Statement 1 is correct.
From statement 2, We know that, 1 x 2 x 3 = 1+ 2+ 3 = 6 .Statement 2 is also correct.
(CSAT 2018)
While writing all the numbers from 700 to 1000, how many numbers occur in which the digit at hundred’s
place is greater than the digit at ten’s place, and the digit at ten’s place is greater than the digit at unit’s
place?
(a) 61
(b) 64
(c) 85
(d) 91
Solution (c):
Given, digit at hundred’s place > digit at tens’ place > digit at unit place. In 700-799 numbers that satisfies the
given conditions are
(710) (720, 721) (730, 731, 732) … (760 – 765)
+1 +2 +3 +6
It would be = 21.
Similarly for 800-899. It would be 1 + 2 + 3 + … 7 = 28
(CSAT 2016)
How many numbers are there between 100 and 300 which either begin with or end with 2?
(a) 110
(b) 111
(c) 112
(d) None of these
Solution (a):
There are 10 numbers which either being with or end with 2,
i.e. 102, 112, 122, 132, 142, 152, 162, 172, 182, 192
Numbers from 200 to 299 = 100
Total numbers = 100 + 10 = 110
(CSAT 2016)
Four-digit numbers are to be formed using the digits 1, 2, 3 and 4, and none of these four digits are repeated
in any manner. Further,
1. 2 and 3 are not to immediately follow each other
2. 1 is not to be immediately followed by 3.
3. 4 is not to appear at the last place
4. 1 is not to appear at the first place
How many different numbers can be formed?
(a) 6
(b) 8
(c) 9
(d) None of these
Solution (a):
According to the given information, 6 different numbers can be formed,
i.e. 2431, 2143, 3412, 3142, 3421, 4312
(CSAT 2013)
A gardener has 1000 plants: He wants to plant them in such a way that the number of rows and the number
of columns remains the same. What is the minimum number of plants that he needs more for this purpose?
(a) 14
(b) 24
(c) 32
(d) 34
Solution (b):
Let the number of rows and columns be “a”
Then total number of plants = a2
It is clear that 312< 1000 < 322
∴ Hence, 322 -1000 = 1024 -1000 =24 more plants are needed.